Live Discussion!
Open Discussion with Ben Kornell and Andrew Sutherland is going on now!

Summer is a great time to explore cool problems to keep your skills sharp!  Schedule a class today!

G
Topic
First Poster
Last Poster
k a June Highlights and 2025 AoPS Online Class Information
jlacosta   0
Jun 2, 2025
Congratulations to all the mathletes who competed at National MATHCOUNTS! If you missed the exciting Countdown Round, you can watch the video at this link. Are you interested in training for MATHCOUNTS or AMC 10 contests? How would you like to train for these math competitions in half the time? We have accelerated sections which meet twice per week instead of once starting on July 8th (7:30pm ET). These sections fill quickly so enroll today!

[list][*]MATHCOUNTS/AMC 8 Basics
[*]MATHCOUNTS/AMC 8 Advanced
[*]AMC 10 Problem Series[/list]
For those interested in Olympiad level training in math, computer science, physics, and chemistry, be sure to enroll in our WOOT courses before August 19th to take advantage of early bird pricing!

Summer camps are starting this month at the Virtual Campus in math and language arts that are 2 - to 4 - weeks in duration. Spaces are still available - don’t miss your chance to have a transformative summer experience. There are middle and high school competition math camps as well as Math Beasts camps that review key topics coupled with fun explorations covering areas such as graph theory (Math Beasts Camp 6), cryptography (Math Beasts Camp 7-8), and topology (Math Beasts Camp 8-9)!

Be sure to mark your calendars for the following upcoming events:
[list][*]June 5th, Thursday, 7:30pm ET: Open Discussion with Ben Kornell and Andrew Sutherland, Art of Problem Solving's incoming CEO Ben Kornell and CPO Andrew Sutherland host an Ask Me Anything-style chat. Come ask your questions and get to know our incoming CEO & CPO!
[*]June 9th, Monday, 7:30pm ET, Game Jam: Operation Shuffle!, Come join us to play our second round of Operation Shuffle! If you enjoy number sense, logic, and a healthy dose of luck, this is the game for you. No specific math background is required; all are welcome.[/list]
Our full course list for upcoming classes is below:
All classes run 7:30pm-8:45pm ET/4:30pm - 5:45pm PT unless otherwise noted.

Introductory: Grades 5-10

Prealgebra 1 Self-Paced

Prealgebra 1
Sunday, Jun 15 - Oct 12
Monday, Jun 30 - Oct 20
Wednesday, Jul 16 - Oct 29
Sunday, Aug 17 - Dec 14
Tuesday, Aug 26 - Dec 16
Friday, Sep 5 - Jan 16
Monday, Sep 8 - Jan 12
Tuesday, Sep 16 - Jan 20 (4:30 - 5:45 pm ET/1:30 - 2:45 pm PT)
Sunday, Sep 21 - Jan 25
Thursday, Sep 25 - Jan 29
Wednesday, Oct 22 - Feb 25
Tuesday, Nov 4 - Mar 10
Friday, Dec 12 - Apr 10

Prealgebra 2 Self-Paced

Prealgebra 2
Monday, Jun 2 - Sep 22
Sunday, Jun 29 - Oct 26
Friday, Jul 25 - Nov 21
Sunday, Aug 17 - Dec 14
Tuesday, Sep 9 - Jan 13
Thursday, Sep 25 - Jan 29
Sunday, Oct 19 - Feb 22
Monday, Oct 27 - Mar 2
Wednesday, Nov 12 - Mar 18

Introduction to Algebra A Self-Paced

Introduction to Algebra A
Sunday, Jun 15 - Oct 12
Thursday, Jun 26 - Oct 9
Tuesday, Jul 15 - Oct 28
Sunday, Aug 17 - Dec 14
Wednesday, Aug 27 - Dec 17
Friday, Sep 5 - Jan 16
Thursday, Sep 11 - Jan 15
Sunday, Sep 28 - Feb 1
Monday, Oct 6 - Feb 9
Tuesday, Oct 21 - Feb 24
Sunday, Nov 9 - Mar 15
Friday, Dec 5 - Apr 3

Introduction to Counting & Probability Self-Paced

Introduction to Counting & Probability
Sunday, Jun 1 - Aug 24
Thursday, Jun 12 - Aug 28
Wednesday, Jul 2 - Sep 17
Sunday, Jul 27 - Oct 19
Monday, Aug 11 - Nov 3
Wednesday, Sep 3 - Nov 19
Sunday, Sep 21 - Dec 14 (1:00 - 2:30 pm ET/10:00 - 11:30 am PT)
Friday, Oct 3 - Jan 16
Tuesday, Nov 4 - Feb 10
Sunday, Dec 7 - Mar 8

Introduction to Number Theory
Monday, Jun 9 - Aug 25
Sunday, Jun 15 - Sep 14
Tuesday, Jul 15 - Sep 30
Wednesday, Aug 13 - Oct 29
Friday, Sep 12 - Dec 12
Sunday, Oct 26 - Feb 1
Monday, Dec 1 - Mar 2

Introduction to Algebra B Self-Paced

Introduction to Algebra B
Wednesday, Jun 4 - Sep 17
Sunday, Jun 22 - Oct 19
Friday, Jul 18 - Nov 14
Thursday, Aug 7 - Nov 20
Monday, Aug 18 - Dec 15
Sunday, Sep 7 - Jan 11
Thursday, Sep 11 - Jan 15
Wednesday, Sep 24 - Jan 28
Sunday, Oct 26 - Mar 1
Tuesday, Nov 4 - Mar 10
Monday, Dec 1 - Mar 30

Introduction to Geometry
Monday, Jun 16 - Dec 8
Friday, Jun 20 - Jan 9
Sunday, Jun 29 - Jan 11
Monday, Jul 14 - Jan 19
Wednesday, Aug 13 - Feb 11
Tuesday, Aug 26 - Feb 24
Sunday, Sep 7 - Mar 8
Thursday, Sep 11 - Mar 12
Wednesday, Sep 24 - Mar 25
Sunday, Oct 26 - Apr 26
Monday, Nov 3 - May 4
Friday, Dec 5 - May 29

Paradoxes and Infinity
Mon, Tue, Wed, & Thurs, Jul 14 - Jul 16 (meets every day of the week!)

Intermediate: Grades 8-12

Intermediate Algebra
Sunday, Jun 1 - Nov 23
Tuesday, Jun 10 - Nov 18
Wednesday, Jun 25 - Dec 10
Sunday, Jul 13 - Jan 18
Thursday, Jul 24 - Jan 22
Friday, Aug 8 - Feb 20
Tuesday, Aug 26 - Feb 24
Sunday, Sep 28 - Mar 29
Wednesday, Oct 8 - Mar 8
Sunday, Nov 16 - May 17
Thursday, Dec 11 - Jun 4

Intermediate Counting & Probability
Sunday, Jun 22 - Nov 2
Sunday, Sep 28 - Feb 15
Tuesday, Nov 4 - Mar 24

Intermediate Number Theory
Sunday, Jun 1 - Aug 24
Wednesday, Jun 18 - Sep 3
Wednesday, Sep 24 - Dec 17

Precalculus
Sunday, Jun 1 - Nov 9
Monday, Jun 30 - Dec 8
Wednesday, Aug 6 - Jan 21
Tuesday, Sep 9 - Feb 24
Sunday, Sep 21 - Mar 8
Monday, Oct 20 - Apr 6
Sunday, Dec 14 - May 31

Advanced: Grades 9-12

Olympiad Geometry
Tuesday, Jun 10 - Aug 26

Calculus
Wednesday, Jun 25 - Dec 17
Sunday, Sep 7 - Mar 15
Wednesday, Sep 24 - Apr 1
Friday, Nov 14 - May 22

Group Theory
Thursday, Jun 12 - Sep 11

Contest Preparation: Grades 6-12

MATHCOUNTS/AMC 8 Basics
Monday, Jun 2 - Aug 18
Thursday, Jun 12 - Aug 28
Sunday, Jun 22 - Sep 21
Tues & Thurs, Jul 8 - Aug 14 (meets twice a week!)
Sunday, Aug 17 - Nov 9
Wednesday, Sep 3 - Nov 19
Tuesday, Sep 16 - Dec 9
Sunday, Sep 21 - Dec 14 (1:00 - 2:30 pm ET/10:00 - 11:30 am PT)
Monday, Oct 6 - Jan 12
Thursday, Oct 16 - Jan 22
Tues, Thurs & Sun, Dec 9 - Jan 18 (meets three times a week!)

MATHCOUNTS/AMC 8 Advanced
Wednesday, Jun 11 - Aug 27
Sunday, Jun 22 - Sep 21
Tues & Thurs, Jul 8 - Aug 14 (meets twice a week!)
Sunday, Aug 17 - Nov 9
Tuesday, Aug 26 - Nov 11
Thursday, Sep 4 - Nov 20
Friday, Sep 12 - Dec 12
Monday, Sep 15 - Dec 8
Sunday, Oct 5 - Jan 11
Tues, Thurs & Sun, Dec 2 - Jan 11 (meets three times a week!)
Mon, Wed & Fri, Dec 8 - Jan 16 (meets three times a week!)

AMC 10 Problem Series
Sunday, Jun 1 - Aug 24
Thursday, Jun 12 - Aug 28
Tuesday, Jun 17 - Sep 2
Sunday, Jun 22 - Sep 21 (1:00 - 2:30 pm ET/10:00 - 11:30 am PT)
Monday, Jun 23 - Sep 15
Tues & Thurs, Jul 8 - Aug 14 (meets twice a week!)
Sunday, Aug 10 - Nov 2
Thursday, Aug 14 - Oct 30
Tuesday, Aug 19 - Nov 4
Mon & Wed, Sep 15 - Oct 22 (meets twice a week!)
Mon, Wed & Fri, Oct 6 - Nov 3 (meets three times a week!)
Tue, Thurs & Sun, Oct 7 - Nov 2 (meets three times a week!)

AMC 10 Final Fives
Monday, Jun 30 - Jul 21
Friday, Aug 15 - Sep 12
Sunday, Sep 7 - Sep 28
Tuesday, Sep 9 - Sep 30
Monday, Sep 22 - Oct 13
Sunday, Sep 28 - Oct 19 (1:00 - 2:30 pm ET/10:00 - 11:30 am PT)
Wednesday, Oct 8 - Oct 29
Thursday, Oct 9 - Oct 30

AMC 12 Problem Series
Thursday, Jun 12 - Aug 28
Sunday, Jun 22 - Sep 21
Wednesday, Aug 6 - Oct 22
Sunday, Aug 10 - Nov 2
Monday, Aug 18 - Nov 10
Mon & Wed, Sep 15 - Oct 22 (meets twice a week!)
Tues, Thurs & Sun, Oct 7 - Nov 2 (meets three times a week!)

AMC 12 Final Fives
Thursday, Sep 4 - Sep 25
Sunday, Sep 28 - Oct 19
Tuesday, Oct 7 - Oct 28

AIME Problem Series A
Thursday, Oct 23 - Jan 29

AIME Problem Series B
Sunday, Jun 22 - Sep 21
Tuesday, Sep 2 - Nov 18

F=ma Problem Series
Wednesday, Jun 11 - Aug 27
Tuesday, Sep 16 - Dec 9
Friday, Oct 17 - Jan 30

WOOT Programs
Visit the pages linked for full schedule details for each of these programs!


MathWOOT Level 1
MathWOOT Level 2
ChemWOOT
CodeWOOT
PhysicsWOOT

Programming

Introduction to Programming with Python
Sunday, Jun 15 - Sep 14 (1:00 - 2:30 pm ET/10:00 - 11:30 am PT)
Tuesday, Jun 17 - Sep 2
Monday, Jun 30 - Sep 22
Thursday, Aug 14 - Oct 30
Sunday, Sep 7 - Nov 23
Tuesday, Dec 2 - Mar 3

Intermediate Programming with Python
Sunday, Jun 1 - Aug 24
Monday, Jun 30 - Sep 22
Friday, Oct 3 - Jan 16

USACO Bronze Problem Series
Sunday, Jun 22 - Sep 1
Wednesday, Sep 3 - Dec 3
Thursday, Oct 30 - Feb 5
Tuesday, Dec 2 - Mar 3

Physics

Introduction to Physics
Sunday, Jun 15 - Sep 14
Monday, Jun 23 - Sep 15
Tuesday, Sep 2 - Nov 18
Sunday, Oct 5 - Jan 11
Wednesday, Dec 10 - Mar 11

Physics 1: Mechanics
Monday, Jun 23 - Dec 15
Sunday, Sep 21 - Mar 22
Sunday, Oct 26 - Apr 26

Relativity
Mon, Tue, Wed & Thurs, Jun 23 - Jun 26 (meets every day of the week!)
0 replies
jlacosta
Jun 2, 2025
0 replies
Tricky FE
Rijul saini   12
N 20 minutes ago by TestX01
Source: LMAO 2025 Day 1 Problem 1
Let $\mathbb{R}$ denote the set of all real numbers. Find all functions $f : \mathbb{R} \to \mathbb{R}$ such that
$$f(xy) + f(f(y)) = f((x + 1)f(y))$$for all real numbers $x$, $y$.

Proposed by MV Adhitya and Kanav Talwar
12 replies
Rijul saini
Wednesday at 6:58 PM
TestX01
20 minutes ago
Permutation guessing game
Rijul saini   2
N 38 minutes ago by Supercali
Source: India IMOTC Day 3 Problem 3
Let $n$ be a positive integer. Alice and Bob play the following game. Alice considers a permutation $\pi$ of the set $[n]=\{1,2, \dots, n\}$ and keeps it hidden from Bob. In a move, Bob tells Alice a permutation $\tau$ of $[n]$, and Alice tells Bob whether there exists an $i \in [n]$ such that $\tau(i)=\pi(i)$. Bob wins if he ever tells Alice the permutation $\pi$. Prove that Bob can win the game in at most $n \log_2(n) + 2025n$ moves.

Proposed by Siddharth Choppara and Shantanu Nene
2 replies
Rijul saini
Wednesday at 6:43 PM
Supercali
38 minutes ago
One of P or Q lies on circle
Rijul saini   7
N 2 hours ago by MathLuis
Source: LMAO 2025 Day 1 Problem 3
Let $ABC$ be an acute triangle with orthocenter $H$. Let $M$ be the midpoint of $BC$, and $K$ be the intersection of the tangents from $B$ and $C$ to the circumcircle of $ABC$. Denote by $\Omega$ the circle centered at $H$ and tangent to line $AM$.

Suppose $AK$ intersects $\Omega$ at two distinct points $X$, $Y$.
Lines $BX$ and $CY$ meet at $P$, while lines $BY$ and $CX$ meet at $Q$. Prove that either $P$ or $Q$ lies on $\Omega$.

Proposed by MV Adhitya, Archit Manas and Arnav Nanal
7 replies
Rijul saini
Wednesday at 6:59 PM
MathLuis
2 hours ago
Cheese??? - I'm definitely doing smth wrong
Sid-darth-vater   1
N 2 hours ago by Diamond-jumper76
Source: European Girls Math Olympiad 2013/1
The problem is attached. So is my diagram which has a couple of markings on it for clarity :)

So basically, I found a solution which I am 99% confident that I am doing smth wrong, I just can't find the error. Any help would be appreciated!

We claim that triangle $BAC$ is right angled (for clarity, $<BAC = 90$). Define $S$ as a point on line $AC$ such that $SD$ is parallel to $AB$. Additionally, since $BC = DC$, $\triangle BAC \cong \triangle DSC$ meaning $<BAC = <CSD$, $AC = CS$, and $AB = SD$. Also, since $BE = AD$, by SSS, we have $\triangle BEA \cong DAS$ meaning $\angle EAB= \angle CSD$. Since $\angle EAS + \angle BAC = 180$, we have $2\angle ASD = 180$ or $\angle ASD = \angle BAC = 90$ and we are done.
1 reply
Sid-darth-vater
4 hours ago
Diamond-jumper76
2 hours ago
No more topics!
Imo Shortlist Problem
Lopes   35
N Apr 24, 2025 by Maximilian113
Source: IMO Shortlist 2000, Problem N4
Find all triplets of positive integers $ (a,m,n)$ such that $ a^m + 1 \mid (a + 1)^n$.
35 replies
Lopes
Feb 27, 2005
Maximilian113
Apr 24, 2025
Imo Shortlist Problem
G H J
Source: IMO Shortlist 2000, Problem N4
The post below has been deleted. Click to close.
This post has been deleted. Click here to see post.
Lopes
42 posts
#1 • 6 Y
Y by itslumi, FaThEr-SqUiRrEl, Adventure10, Mango247, and 2 other users
Find all triplets of positive integers $ (a,m,n)$ such that $ a^m + 1 \mid (a + 1)^n$.
Z K Y
The post below has been deleted. Click to close.
This post has been deleted. Click here to see post.
Davron
484 posts
#2 • 5 Y
Y by mathisreal, FaThEr-SqUiRrEl, Adventure10, Mango247, and 1 other user
Answer: (a, m, n) = (1, m, n), (a, 1, n) or (2, 3, k) where k > 1.

It is obvious that (1, m, n) and (a, 1, n) are solutions, so assume a > 1 and m > 1. We show first that m must be odd.

Suppose there is an odd prime p dividing am + 1. Then p must also divide (a + 1)n and hence (a + 1). So a = -1 mod p, so am + 1 = (-1)m + 1 mod p. Hence m is odd. If there is no such prime p, then am + 1 = 2k for some k. But a and m are assumed to be at least 2, so am + 1 must be at least 5 and hence k ≥ 3. So am = -1 mod 4. But squares can only be 0 or 1 mod 4, so am cannot be a square, so m must be odd.

Suppose p divides m. Then (since m is odd) ap + 1 divides am + 1 and hence (a + 1)n. But p is odd, so a + 1 divides ap + 1 and (ap + 1)/(a + 1) = ap-1 - ap-2 + ap-3 - ... - p + 1 divides (a + 1)n-1. Put f(x) = xp-1 - xp-2 + ... - x + 1. Then f(-1) = p, so f(x) - p = (x + 1) g(x) for some polynomial g(x). Putting x = a, we get f(a) = (a + 1) g(a) + p. If a prime q divides f(a), then since f(a) divides (a + 1)n-1, q must divide (a + 1). Hence q divides p, so q = p. So f(a) = pk for some k ≥ 1. Since f(a) divides (a + 1)n-1, p must divide a + 1.

We show next that k = 1, so f(a) = p. If p2 divides (a + 1), then f(a) = (a + 1) g(a) + p implies that p2 does not divide f(a). If p2 does not divide (a + 1), then (a + 1) = ph (with h not divisible by p). So f(a) = (ap + 1)/(a + 1) = ( (1 + ph - 1)p)/ph = ( 1 - 1 + p2h + terms divisible by p3)/ph = p mod p2, so again p2 does not divide f(a). So in any case f(a) = p.

Now (a2 - a) ≥ 2, (a4 - a3) ≥ 8 and obviously (a2r - a2r-1) ≥ 2 for r ≥ 2. So f(a) ≥ 11 > 5 for p = 5, and hence by a trivial induction f(a) > p for p > 5. So the only possibility is p = 3. Then we have a2 - a + 1 = 3 and hence a = 2.

So far we have established that a = 2 and that m is a power of 3. But (29 + 1) = 513, which is not a power of 3 and so cannot divide (2 + 1)n. But (227 + 1), (281 + 1), ... are all divisible by (29 + 1) and hence are also not powers of 3. So m must be 3. Finally, we require that (am + 1) = 9 divides (2 + 1)n, so n ≥ 2.

davron ft. kalva
Z K Y
The post below has been deleted. Click to close.
This post has been deleted. Click here to see post.
trigg
36 posts
#3 • 3 Y
Y by FaThEr-SqUiRrEl, Adventure10, Mango247
I think this problem is not hard but it is nice :)
Z K Y
The post below has been deleted. Click to close.
This post has been deleted. Click here to see post.
ZetaX
7579 posts
#4 • 3 Y
Y by FaThEr-SqUiRrEl, Adventure10, Mango247
Like the similar ones just a corollary of Zsigmondy's theorem :P
Z K Y
The post below has been deleted. Click to close.
This post has been deleted. Click here to see post.
Scientist
11 posts
#5 • 2 Y
Y by FaThEr-SqUiRrEl, Adventure10
If a=1 then 〖 1〗^m+1|〖(1+1)〗^n this means for all m,n∈Z^+ and a=1 is solution. If a=2 and m=3 then 〖 2〗^3+1|3^n this means a=2 m=3 and for all n≥2 and n∈Z^+ is solution.If m=1 then for all a,n∈Z^+ is solution. If (a,m)≠(2,3) and a≠1 and m≠1 then from the Zsigmondy theorem ∃p∈prime p|a^m+1 but p∤a+1 this means contradiction.
Z K Y
The post below has been deleted. Click to close.
This post has been deleted. Click here to see post.
quantumbyte
547 posts
#6 • 6 Y
Y by Max D.R., nguyendangkhoa17112003, FaThEr-SqUiRrEl, Adventure10, Mango247, and 1 other user
Quick way to start the proof before some cases:
Solution outline
Z K Y
The post below has been deleted. Click to close.
This post has been deleted. Click here to see post.
sjaelee
485 posts
#7 • 3 Y
Y by FaThEr-SqUiRrEl, DCMaths, Adventure10
If $m$ is even, the any prime $p|a+1$ or $a\equiv -1\pmod{p}$ and $a^m+1\equiv 2\pmod{p}$. If $a+1$ is a power of $2$, then we have a contradiction as primes other than $2$ divide $a^m+1$ unless $a^m+1=2$, which means $a=1$ and discounts the case otherwise.

For $m$ odd, $a+1 | a^m+1$ and any prime divisor of $a^m+1$ divides $(a+1)^n$, implying all prime divisors of $a^m+1$ divides $a+1$. By LTE if a prime $p$ divides $a+1$, then $v_p(a^m+1)=v_p(a+1)+v_p(m)$. Therefore if all the prime divisors of $a+1$ are the prime divisors of $a^m+1$, and the $v_p$ of any prime dividing $a^m+1$ divides $m$, $\frac{a^m+1}{a+1} | m$.

If $m=1$ obviously all $a$ and $n$ work. If $a=1$ clearly all $m,n$ work. If $m>3$ the $\frac{a^m+1}{a+1}>m$ for $a\ge 2$. If $m=3$ the only solution is $a=2$ otherwise $\frac{a^m+1}{a+1}>m$.
Z K Y
The post below has been deleted. Click to close.
This post has been deleted. Click here to see post.
sayantanchakraborty
505 posts
#8 • 7 Y
Y by Sx763_, FaThEr-SqUiRrEl, ZHEKSHEN, NVA9205, Adventure10, Mango247, panche
It is easy to note that $(a,m,n)=(1,m,n)$ is a solution.Let $a \ge 2$.Also suppose $m \ge 2$ and $(a,m) \neq (2,3)$.Then by Zsigmondy's theorem there exists a primitive prime factor of $a^m+1$ which does not divide $(a+1)$ and hence doesn't divide $(a+1)^n$.For $m=1$ we get the trivial solution $(a,1,n)$.For $a=2,m=3$ the condition is true for $n \ge 2$ so $(2,3,n \ge 2)$ are also solutions.
Z K Y
The post below has been deleted. Click to close.
This post has been deleted. Click here to see post.
tenplusten
1000 posts
#9 • 2 Y
Y by FaThEr-SqUiRrEl, Adventure10
Too easy problem.
$(a,m,n)=(1,m,n),(2,3,n)$
It is obvious that $m$ is odd. İf $m$ is even:
Take $p|a^m+1$ then $p|a+1$ or $a\equiv -1 mod p$ $a^m+1\equiv (-1)^m+1 mod p$. Contradiction.
Claim: $a^m+1$ and $a+1$ have the same prime divisors.
Proof:Take an arbitrary $p|a^m+1$ then $p|(a+1)^n$ since $p$ is prime $p|a+1$.

By Zsigmondy's theorem there is $q|a^m+1$ but not $q|a+1$.Contradiction.

Comment
This post has been edited 2 times. Last edited by tenplusten, Mar 24, 2017, 5:10 AM
Z K Y
The post below has been deleted. Click to close.
This post has been deleted. Click here to see post.
ubermensch
820 posts
#10 • 3 Y
Y by FaThEr-SqUiRrEl, Adventure10, Mango247
Wait Zsigmondy was still so unpopular even as late as $2000$? (as this is not even a one-liner using Zsigmondy and its exceptions)...
Z K Y
The post below has been deleted. Click to close.
This post has been deleted. Click here to see post.
AlastorMoody
2125 posts
#12 • 1 Y
Y by FaThEr-SqUiRrEl
Solution
Z K Y
The post below has been deleted. Click to close.
This post has been deleted. Click here to see post.
R-sk
429 posts
#13 • 1 Y
Y by FaThEr-SqUiRrEl
Here since we can easily find the trivial solutions then by zsigmondy there exist a divisor of a^m+1 that does not divide a+1 and therefore does not divide (a+1) ^n and therefore there are no other solutions other than (1, m, n), (2, 3,n), (a, 1,n)
Z K Y
The post below has been deleted. Click to close.
This post has been deleted. Click here to see post.
MatBoy-123
396 posts
#14 • 1 Y
Y by FaThEr-SqUiRrEl
N5 is also an application of Zsimondy's..
Z K Y
The post below has been deleted. Click to close.
This post has been deleted. Click here to see post.
isaacmeng
113 posts
#15 • 1 Y
Y by FaThEr-SqUiRrEl
This can be trivialized by Zsigmondy's.

ISL 00N4. Find all $(a, m, n)\in\mathbb{N}^3$ such that $a^m+1\mid (a+1)^n$.

Solution. If $a=1$ then the divisibility condition obviously work. Assume now that $a\ge 2$.
If $m=1$ then the divisibility condition obviously work. Assume now that $a\ge 2$, $m\ge 2$.
If the two conditions $a=2$, $m=3$ are not both true then by Zsigmondy's there must exist a prime factor $p$ of $a^m+1$ which does not divide $a+1$ (and of course so does not $(a+1)^n$). Hence there is no solution in this case.
It suffices for us to consider the single case $a=2$, $m=3$, in which only all $n\ge 2$ satisfy the divisibility condition.
Therefore the solutions are only $(1, m, n), (a, 1, n), (2, 3, k)$ where $k\ge 2$.
This post has been edited 4 times. Last edited by isaacmeng, Apr 20, 2021, 1:30 PM
Z K Y
The post below has been deleted. Click to close.
This post has been deleted. Click here to see post.
hakN
429 posts
#16 • 4 Y
Y by FaThEr-SqUiRrEl, Mango247, Mango247, Mango247
If $a=1$ then we get $(1,m,n)$ as solutions for all $m,n\in \mathbb{N}$.
If $a=2$ then we get that $2^m + 1\mid 3^n$. If $m=1$ then we get $(2,1,1)$ as a solution. If $m\geq 2$, then $2^m + 1 = 3^k$ doesnt have any solution except $(m,k)=(3,2)$ becuase of Mihailescu's Theorem.

Let $a\geq 3$ and $m\geq 2$. By Zsigmondy, $a^m + 1$ has a prime divisor that doesnt divide $a+1$. Thus, if we take that divisor we clearly get no solution. If $m=1$ then we get $(a,1,n)$ as solutions.
So all solutions are $(1,m,n) , (a,1,n) , (2,3,n)$.
This post has been edited 2 times. Last edited by hakN, Apr 20, 2021, 6:43 PM
Z K Y
The post below has been deleted. Click to close.
This post has been deleted. Click here to see post.
oVlad
1746 posts
#17
Y by
Here's another way to solve this without Zsigmondy:

If $p\mid a^m+1$ then $p\mid (a+1)^n$ so $p\mid a+1$ so $$\nu_p(a^m+1)=\nu_p(a+1)+\nu_p(m)\leq \nu_p(a+1)\big(\nu_p(m)+1\big)\leq \nu_p(a+1)\big(\log_2 m+1\big)$$Hence, we gat that $a^m+1\mid (a+1)^{\log_2 m+1}$ so $(a+1)^{\log_2 m+1}\geq a^m+1.$ However, by Holder's Inequality, we get that \[2^{\log_2 m+1}\big(a^{\log_2 m+1}+1\big)\geq (a+1)^{\log_2 m+1}\geq a^m+1\]so by letting $\log_2 m=c$ we essentially got that $2^{c+1}\big(a^{c+1}+1\big)\geq a^{2^c}+1$ which, after some easy bounding, will give us $a=1, m=1$ or $a,m\leq 3.$
This post has been edited 2 times. Last edited by oVlad, Oct 21, 2021, 9:21 AM
Z K Y
The post below has been deleted. Click to close.
This post has been deleted. Click here to see post.
MathLuis
1562 posts
#18
Y by
Sorry for using Zsigmondy when almost everyone did :blush: , i just couldnt resist the free problem xD

Case 1: $m=1$
Then $a+1 \mid (a+1)^n$ which holds for any $a,n \in \mathbb N$ thus $(a,m,n)=(a,1,n)$
Case 2: $a=1$
Then $2 \mid 2^n$ which holds for any $m,n \in \mathbb N$ thus $(a,m,n)=(1,m,n)$
Case 3: $a,m>1$
Since $\gcd(a,1)=1$ we can use Zsigmondy assuming that $(a,m) \ne (2,3)$ so let $p$ such that $p \mid a^m+1$ and $p \not \; \mid a+1$ but since $p \mid a^m+1 \mid (a+1)^n$ we get a contraidiction.
Thus $a=2$ and $m=3$ and since $9 \mid 3^n$ holds for $n \ge 2$ the sol is $(a,m,n)=(2,3,n)$ where $n>2$

Thus we are done :blush:
Z K Y
The post below has been deleted. Click to close.
This post has been deleted. Click here to see post.
Mogmog8
1080 posts
#19 • 1 Y
Y by centslordm
By Zsigmondy, $a^m+1$ has a primitive prime factor if $(a,m)\neq(2,3);$ let the prime be $p.$ If $m>1$ and $a>1,$ $p\nmid a+1,$ so $a^m+1\nmid (a+1)^n.$ If $a=1$ or $m=1,$ notice that $1+1\mid 2^n$ and $a+1\mid(a+1)^n,$ respectively. If $(a,m)=(2,3)$ and $n>1,$ we have $2^3+1\mid(2+1)^n,$ so our solutions are $\boxed{(a,1,n),(1,m,n)},$ and $\boxed{(2,3,n)\text{ where }n>1}.$ $\square$
Z K Y
The post below has been deleted. Click to close.
This post has been deleted. Click here to see post.
hydo2332
435 posts
#20
Y by
We divide in two cases.

1) $m$ is even.
In this case, we have that if $p \mid a+1$, then $p \mid a^m + 1$, therefore $p \mid 2$ since $m$ is even. So we get $a^m + 1 = 2^x$ for some $x$, but notice that $4 \nmid a^m + 1$ so we conclude $x=1, a=1$. In this case we get the solution $(a,n,m) = (a,2m',n)$ for any $m',n$ positive integers.

2)$m$ is odd. In this case, we have that $a+1 \mid a^m + 1$. So we get that $\dfrac{a^m + 1}{a+1} \mid (a + 1)^{n-1}$. Notice that $\dfrac{a^m + 1}{a+1} = a^{m-1} - a^{m-2} + \dots + 1$ is odd since $m$ is odd and therefore it has no factors $2$.

So we can apply the LTE Lemma, which concludes that $\nu_p(\dfrac{a^m + 1}{a+1}) = \nu_p(m)$ for every prime $p$ that divides $\dfrac{a^m + 1}{a+1}$ (which are almost all the same primes that divide $a^m + 1$, the same as the ones that divide $a+1$). But this clearly implies that $\dfrac{a^m + 1}{a+1} \mid m$. Looking at the size of the left hand size, we get that either $a=1$, $m=3$ or $m=1$. If $a=1$, we get the solution $(a,n,m) = (1,n,2m'+1)$ for any $m',n$ positive integers. If $m=3$, we either have $a=2$ or $a=1$. If $a=1$ we are already done, and if $a=2$ we get that $n=2$ so we get the solution $(a,n,m) = (2,2,3)$. Finally, when $m=1$ we get the solution $(a,n,m) = (a,n,1)$ for any $a,n$ positive integers.

Finally, we get that the solutions are $(a,n,m) = (2,2,3), (1,n,m), (a,n,1)$.
This post has been edited 1 time. Last edited by hydo2332, Apr 1, 2022, 3:03 AM
Z K Y
The post below has been deleted. Click to close.
This post has been deleted. Click here to see post.
awesomeming327.
1746 posts
#21
Y by
oof^tm

Note that $(a,1,n)$, $(1,m,n)$ and $(2,3,n),n\ge 2$ are solutions. We claim that these are the only solutions. Suppose $a,m>1.$ Then, $n=1$ clearly doesn't work so $n>1$ as well. Also remove case $(2,3,n).$ By Zsigmondy's there exists a prime factor of $a^m+1$ that doesn't divide $a+1$ so it doesn't divide $(a+1)^n$ as well. Thus, we're done.
Z K Y
The post below has been deleted. Click to close.
This post has been deleted. Click here to see post.
megarnie
5610 posts
#22
Y by
The only solutions are $\boxed{(1,m,n), (a,1,n), (2,3,n+1)}$, where $a$, $m$, and $n$ are any positive integers.

The first one works because the LHS is $2$, the RHS is $2^n$, and $2\mid 2^n$.

The second one works because the LHS is $a+1$, the RHS is $(a+1)^n$, and $a+1\mid (a+1)^n$.

The third one works because the LHS is $9$, the RHS is $3^{n+1}$. Since $n+1\ge 2$, $9\mid 3^n$.


It remains to show that those are the only solutions.

If $a=1$, we get the solution $(1,m,n)$.

If $m=1$, we get the solution $(a,1,n)$.

Now suppose $a>1$ and $m>1$.

If $a=2$ and $m=3$, then we have $9\mid 3^n$, so $n\ge 2$, which gives the solution $(2,3,n+1)$.

Otherwise, by Zsigmondy's, there exists a prime divisor of $a^m+1$ that doesn't divide $a+1$, contradiction.
Z K Y
The post below has been deleted. Click to close.
This post has been deleted. Click here to see post.
HamstPan38825
8880 posts
#23 • 2 Y
Y by awesomeming327., Ritwin
I suppose here's a more instructive solution that isn't some variant of "trivial by Zsigmondy's."

The answers are $(1, m, n)$, $(2, 3, n)$, and $(a, 1, n)$, which work. Henceforth assume that $a, m > 1$ and $a \neq 2$. It suffices to show that $a^m+1$ contains a prime factor that does not divide $a+1$ for all choices of $a$ and $m$.

Observe that it suffices to show the case for $m$ a prime number $p$, as $$\nu_p(a^m + 1) = \nu_p(a^{m/k} + 1) + \nu_p\left(\frac mk\right) > 0$$for any $p \mid a^{m/k} + 1$ by choosing $\frac mk$ prime. In particular, we may choose $p \nmid a+1$ because the hypothesis holds for $\frac mk$ prime.

Now let $m$ be a prime number $p_1$. For any $p \neq p_1$ that divides $a+1$, $$\nu_p(a^{p_1} + 1) = \nu_p(a+1) + \nu_p(p_1) = \nu_p(a+1)$$by LTE, and furthermore if $p_1$ also divides $a+1$, $\nu_{p_1}(a^{p_1} + 1) = 2$ in this case as well. Suppose that $m$ has no prime factors that do not divide $n$; then, $$a^m+1 \leq (a+1)^2$$with equality holding when $a = p_1$ is prime and $m=p_1$. This forces $m=1, 2$ (and also the weird case $m=3, a=2$), from where we can check the cases manually. Otherwise, there must exist some $p$ that divides $a^m+1$ but not $n$, so the hypothesis is true and we are done.
This post has been edited 1 time. Last edited by HamstPan38825, Jul 20, 2022, 4:02 PM
Z K Y
The post below has been deleted. Click to close.
This post has been deleted. Click here to see post.
cj13609517288
1930 posts
#24
Y by
Solved with a hint from Ritwin.

For this to be possible, for all $p$ such that $p|a^m+1$, it must also have $p|(a+1)^n\Rightarrow p|a+1$. However, that is impossible by a variation of Zsigmondy's that says that $a^n+b^n$ has at least one primitive prime divisor with exceptions $2^3+1^3$ and trivial cases. The trivial cases are when $a=1$ and when $m=1$. Therefore, our answers are $(2,3,n)$ where $n\ge2$, $(1,m,n)$ for positive integers $m,n$, and $(a,1,n)$ for positive integers $a,n$.
This post has been edited 1 time. Last edited by cj13609517288, Aug 13, 2022, 5:24 PM
Z K Y
The post below has been deleted. Click to close.
This post has been deleted. Click here to see post.
Msn05
39 posts
#25 • 2 Y
Y by Ibrahim_K, teomihai
The answer is $(a,1,n), (1,m,n)$ and $(2,3,n)$ for $n \geq 2$.
Case 1: $m$ is even.
Notice we want to get $\gcd(a^m+1,(a+1)^n)=a^m+1$ but we have at most $\gcd(a^m+1,(a+1)^n)=2$ (When $a$ is odd). So, $a=1$ since $a \geq 2$ is impossible.
From here we get $(a,m,n)=(1,m,n)$ (Also works for all $m$).
Case 2: $m$ is odd.
$m=1$ obviously works. So from here $(a,m,n)=(a,1,n)$.
Now assume $m>1$. We want to get $\gcd(a^m+1,(a+1)^n)=a^m+1$ but we have $\gcd(a^m+1,(a+1)^n)=\gcd(m(a+1),(a+1)^n)$. After proving by induction we have $a^m+1 \geq am+m$ for all $a \geq 2$ and $m\geq 3$ so we need to solve $m(a+1)=a^m+1$ and from here we get $m \mid a^m+1$. So, $a$ and $m$ are coprime and thus we have $a^m \equiv -1 \pmod{m}$, $a^{2m} \equiv 1 \pmod{m}$ and $a^{\varphi(m)} \equiv 1 \pmod{m}$. So
from Zsigmondy's Theorem we get $a=2,m=3$. And thus we have $(a,m,n)=(2,3,n)$ for $n \geq 2$.
Z K Y
The post below has been deleted. Click to close.
This post has been deleted. Click here to see post.
teomihai
2967 posts
#26
Y by
Msn05 wrote:
The answer is $(a,1,n), (1,m,n)$ and $(2,3,n)$ for $n \geq 2$.
Case 1: $m$ is even.
Notice we want to get $\gcd(a^m+1,(a+1)^n)=a^m+1$ but we have at most $\gcd(a^m+1,(a+1)^n)=2$ (When $a$ is odd). So, $a=1$ since $a \geq 2$ is impossible.
From here we get $(a,m,n)=(1,m,n)$ (Also works for all $m$).
Case 2: $m$ is odd.
$m=1$ obviously works. So from here $(a,m,n)=(a,1,n)$.
Now assume $m>1$. We want to get $\gcd(a^m+1,(a+1)^n)=a^m+1$ but we have $\gcd(a^m+1,(a+1)^n)=\gcd(m(a+1),(a+1)^n)$. After proving by induction we have $a^m+1 \geq am+m$ for all $a \geq 2$ and $m\geq 3$ so we need to solve $m(a+1)=a^m+1$ and from here we get $m \mid a^m+1$. So, $a$ and $m$ are coprime and thus we have $a^m \equiv -1 \pmod{m}$, $a^{2m} \equiv 1 \pmod{m}$ and $a^{\varphi(m)} \equiv 1 \pmod{m}$. So
from Zsigmondy's Theorem we get $a=2,m=3$. And thus we have $(a,m,n)=(2,3,n)$ for $n \geq 2$.

BEAUTIFUL TREATMENT OF CASES!
Z K Y
The post below has been deleted. Click to close.
This post has been deleted. Click here to see post.
math_comb01
662 posts
#27
Y by
Overkill
Solution
Z K Y
The post below has been deleted. Click to close.
This post has been deleted. Click here to see post.
hectorleo123
347 posts
#28
Y by
Lopes wrote:
Find all triplets of positive integers $ (a,m,n)$ such that $ a^m + 1 \mid (a + 1)^n$.
If $a=1\Rightarrow (a,m,n)=(1,m,n)$ is a solution
If $a>1:$
If $m=1 \Rightarrow (a,m,n)=(a,1,n)$ is a solution
If $m>1:$
By Zsigmondy's Theorem:
$\exists$ prime $p/ p|a^m+1, p\nmid a+1$
$\Rightarrow p|(a+1)^n \Rightarrow p|a+1 (\Rightarrow \Leftarrow)$
But there is an exception$(m=3, a=2)$:
$\Rightarrow 9|3^n \Rightarrow n>1$
$\Rightarrow (a,m,n)=(2,3,n), \forall n>1$ is a solution
$\Rightarrow (a,m,n)=(1,m,n),(a,1,n), ((2,3,n),\forall n>1)$ are all solutions
Z K Y
The post below has been deleted. Click to close.
This post has been deleted. Click here to see post.
AleMM
17 posts
#29
Y by
Hello again! :welcomeani:
I know this is a direct application of Zsigmondy's theorem (with base cases exception, of course), but I'd like to present other (instructive) solution.
Any flaws or misstypes that you find you can tell me, I'd like to know. Any kind of constructive feedback is welcomed!


Solution:

Let $a \sim b$ denote that $a$ has the same prime divisors as $b$ and vice versa, that's the same as saying that for a prime $p$, $p|a \iff p|b$.

Lemma: $\boxed{a^m+1 \sim a+1}$
First note that if $a^m+1 \sim a+1$, then writing $a+1=p_1^{\alpha_1} \dots p_k^{\alpha_k}$ and $a^m+1=p_1^{\beta_1} \dots p_k^{\beta_k}$ we just take $n \geq \dfrac{\beta_i}{\alpha_i} \implies \alpha_i n \geq \beta_i \implies a^m+1|(a+1)^n$ (because every prime power in $(a+1)^n$ that is $\alpha_i n$ would be greater than $\beta_i$ due to how we defined $n$), so if $(a, m)$ satisfy $a^m+1 \sim a+1$ it tells us there are infinitely many suffuciently large $n$ for which $a^m+1|(a+1)^n$. So in fact $(a, m)$ such that $a^m+1 \sim a+1$ give us solutions, we just need to prove that all $(a,m)$ solutions are of that form. Let $p$ be any prime divisor of $a^m+1 \implies p|a^m+1|(a+1)^n \implies p|(a+1)^n \implies p|a+1$, so all prime divisors of $a^m+1$ appear in $a+1$ as well. Suppose $m$ is even, write it as $m=2m_0$ and take a prime divisor $p$ of $a^{2m_0}+1 \implies p|a^{2m_0}+1|(a+1)^n \implies p|a+1 \implies a \equiv -1 \pmod{p} \implies a^{2m_0} \equiv (-1)^{2m_0} \equiv -1 \pmod{p}$, but $(-1)^{2m_0}=1 \implies 1 \equiv -1 \pmod{p} \implies p|1-(-1)=2 \implies p=2$. It means that every prime divisor of $a^{2m_0}+1$ is 2, so it's a power of 2 $\implies a^{2m_0}+1 = 2^k$ for some $k \geq 1$. If $k = 1 \implies a=1$ and in fact $\boxed{(1,m,n)}$ works for all positive integers $m,n$. Now, suppose $k \geq 2 \implies 4|2^k \implies 4|a^{2m_0}+1 \implies (a^{m_0})^2 \equiv -1 \pmod{4}$, but -1 isn't a quadratic residue modulo 4! So $m$ is odd $\implies a^m+1=(a+1)(a^{m-1}-a^{m-2}+ \dots -a+1) \implies a+1|a^m+1$, so every prime divisor of $a+1$ must be a prime divisor of $a^m+1$ as well $\square$

Suppose $m > 1$ because $\boxed{(a,1,n)}$ works for $a,n$ positive integers. As we have $a^m+1 \sim a+1$, that is $p|a^m+1 \iff p|a+1$, but $p|a+1 \implies p|a^m+1$, so we will only use that $p|a^m+1 \implies p|a+1$. Choose $p$ to be a prime divisor of $(a^{m-1}-a^{m-2}+ \dots -a+1)$, but because $a^m+1=(a+1)(a^{m-1}-a^{m-2}+ \dots -a+1) \implies p|a^m+1 \implies p|a+1 \implies a \equiv -1 \pmod{p}$ $\implies$ $a^{m-1}-a^{m-2}+ \dots -a+1 \equiv (-1)^{m-1}-(-1)^{m-2}+ \dots -(-1)+1 \equiv \underbrace{1+1+ \dots +1+1}_{m \text{ times}} \pmod{p}$ because $m$ is odd so $m-1$ is even and $m-2$ is odd... But $a^m+1 \equiv 0 \pmod{p} \implies m \equiv 0 \pmod{p}$ for all prime divisors $p$ of $(a^{m-1}-a^{m-2}+ \dots -a+1)$. Which means that $p|(a^{m-1}-a^{m-2}+ \dots -a+1) \implies p|m$. But notice that $p|(a^{m-1}-a^{m-2}+ \dots -a+1) \implies p|a+1$ because $a^m+1 = (a+1)(a^{m-1}-a^{m-2}+ \dots -a+1) \sim a+1$, so by the LTE lemma we have that $V_p(a^{m-1}-a^{m-2}+ \dots -a+1)=V_p(\dfrac{a^m+1}{a+1})=(V_p(a+1)+V_p(m))-V_p(a+1)=V_p(m)$ because $p|a+1$, so every prime divisor $p$ of $(a^{m-1}-a^{m-2}+ \dots -a+1)=\dfrac{a^m+1}{a+1}$ is in $m$ and $V_p(\dfrac{a^m+1}{a+1})=V_p(m)$, thus $\dfrac{a^m+1}{a+1}|m$.

Now we just do case work because $\dfrac{a^m+1}{a+1}|m \implies \dfrac{a^m+1}{a+1} \leq m$ and $\dfrac{a^m+1}{a+1}=(a^{m-1}-a^{m-2}+ \dots -a+1)$ is growing as $a$ grows, so $\dfrac{a^m+1}{a+1} \geq \dfrac{2^m+1}{3} \implies 3m-1 \geq 2^m$, but $2^m > 3m-1$ for $m \geq 4$ and $m$ is odd, so $m=3$ ($m=1$ already seen) $\implies n \geq 2$ and in fact $\boxed{(2,3,n)}$ works for all integers $n \geq 2$. If $a=3 \implies 4m-1 \geq 3^m$ but for $m \geq 2$ we have that $3^m > 4m-1$ and if $a \geq 4$ we get $m \leq 1$, finishing cases. :D
This post has been edited 2 times. Last edited by AleMM, Jul 11, 2023, 6:35 PM
Z K Y
The post below has been deleted. Click to close.
This post has been deleted. Click here to see post.
SatisfiedMagma
462 posts
#30
Y by
Yeah sure, I agree Zsigmondy is good. But you know what's better? Mihăilescu's Theorem and bit of divisibility! The thing is, I was stuck on the last diophantine, but then someone told me this overkill theorem...

Solution: The answers are
  • $(a,m,n) = (1,m,n)$;
  • $(a,m,n) = (a,1,n), (2,3,n)$ for any positive integer $n$.
We will begin by showing that even values of $m$ mostly don't work.

Claim: For even $m$, the only solution is $(a,m,n) = (1,m,n)$ for any $m,n$.

Proof: Let $p$ be some prime divisor of $a^m+1$. Trivially, we get that $p \mid a + 1$. For an even $m$, this would imply that $p \mid (-1)^2 + 1 = 2$. Altogether, one gets that $a+1$ is a power of $2$ and so must be $a^m + 1$.

Write $a = 2^{\alpha} - 1$ for $\alpha \in \mathbb{Z}^+$. Then we have that
\[a^m + 1 = (2^\alpha - 1)^m + 1 = 2^\beta\]with $\beta \in \mathbb{Z}^+$ and $\beta > \alpha$. Reducing the above equation modulo $2^\alpha$, we get $\alpha = 1 \implies a = 2$. Plugging everything back one can see the claim holds true. $\square$
The rest of the solution will deal with the harder case when $m$ is an odd integer. For this, write $a = p^k\cdot l - 1$ for some positive integer $l$ such that $\nu_p(a+1) = k$.

Plugging this into the original relation, one would get
\[(p^kl - 1)^m + 1 \mid (p^kl)^n.\]By Lifting the Exponent Lemma, we can check that $(p^kl - 1)^m + 1$ always divide $p^k$. Since $p \nmid l$, we must have that that
\[(p^kl - 1)^m + 1 \mid l^n.\]Invoking the Binomial Theorem on the LHS, we would get that
\[l(z) + p^k \mid l^{n-1}\]for some some integer $z$. If $l \ne 1$ or $n \ne 1$, then pick some prime divisor $t$ of $l$ such that $t \mid lz + p^k$. But it would be immediately a contradiction. This means either of $l$ or $n$ must be unity.

The case for $n = 1$, can be easily checked out by hand. The final non-trivial case is for $l = 1$. As $m = 1$ clearly works, assume $m > 1$ henceforth. and the rest is easy. For this see that there exists some positive integer $e$ such that
\[(p^k-1)^m + 1 = p^e.\]You can immediately kill this with Mihăilescu's Theorem and suddenly, the solution is complete! $\blacksquare$
Z K Y
The post below has been deleted. Click to close.
This post has been deleted. Click here to see post.
huashiliao2020
1292 posts
#31
Y by
Note that $(a,1,n)$, $(1,m,n)$ and $(2,3,n)\forall n\ge2$ are solutions. We claim that these are the only solutions. Suppose $a,m>1.$ Then, $n=1$ clearly doesn't work so $n>1$ as well. Also remove case $(2,3,n).$

By Zsigmondy a prime dividing a^m+1 doesn't divide a+1 so it doesn't divide (a+1)^n. $\blacksquare$
Z K Y
The post below has been deleted. Click to close.
This post has been deleted. Click here to see post.
KOMKZ
50 posts
#32 • 1 Y
Y by Adil1
I will prove with another way lets take for some $m$ we have $a^m+1$ have all prime divisors $a+1$ and dont have another divisors so $a^m+1$ and $(a+1)^m$ have similar divisors and its will kill the problem by binomial theorem
This post has been edited 1 time. Last edited by KOMKZ, Nov 30, 2023, 3:11 AM
Z K Y
The post below has been deleted. Click to close.
This post has been deleted. Click here to see post.
Eka01
204 posts
#33
Y by
Storage
Z K Y
The post below has been deleted. Click to close.
This post has been deleted. Click here to see post.
quantam13
126 posts
#34
Y by
Storage: Trivial by zsigmondy
Z K Y
The post below has been deleted. Click to close.
This post has been deleted. Click here to see post.
HasnatFarooq
33 posts
#35
Y by
$(a,1,n),(1,m,n)$ are trivial solutions. Suppose, $a,m>1$ then by Zsigmondy theorem there exist a prime $p$ such that $p \nmid a+1$ with the exception being $(a,m)=(2,3)$ which is another solution.
This post has been edited 1 time. Last edited by HasnatFarooq, Feb 25, 2025, 7:49 AM
Reason: .
Z K Y
The post below has been deleted. Click to close.
This post has been deleted. Click here to see post.
ray66
49 posts
#36
Y by
Suppose that $a,m$>1. Then there are no solutions because there exists a prime $p$ that divides $a^m+1$ but not $a+1$ by Zsigmondy. The exception is $(2,3)$, and plugging this in gives $(2,3,n)$ as a solution. $a=1$ is also a solution, and $m=1$ is another. So the only solutions are $(1,m,n)$, $(m,1,n)$, and $(2,3,n)$
Z K Y
The post below has been deleted. Click to close.
This post has been deleted. Click here to see post.
Maximilian113
578 posts
#37
Y by
If $a>2, m > 1$ by Zsigmondy's there is no solution. If $a > 2, m=1$ clearly $n$ can take any value so $(a, 1, n)$ works. If $a=2,$ by Zsigmondy's $m=3$ and then $n \geq 2$ suffices. So $(2, 3, k)$ where $k \geq 2$ works. Finally if $a=1$ we clearly have $(1, m, n)$ so the solutions (which clearly work) are $(a, 1, n), (2, 3, k), (1, m, n)$ with $k \geq 2.$
Z K Y
N Quick Reply
G
H
=
a